Stay ahead of learning milestones! Enroll in a class over the summer!

G
Topic
First Poster
Last Poster
k a April Highlights and 2025 AoPS Online Class Information
jlacosta   0
Apr 2, 2025
Spring is in full swing and summer is right around the corner, what are your plans? At AoPS Online our schedule has new classes starting now through July, so be sure to keep your skills sharp and be prepared for the Fall school year! Check out the schedule of upcoming classes below.

WOOT early bird pricing is in effect, don’t miss out! If you took MathWOOT Level 2 last year, no worries, it is all new problems this year! Our Worldwide Online Olympiad Training program is for high school level competitors. AoPS designed these courses to help our top students get the deep focus they need to succeed in their specific competition goals. Check out the details at this link for all our WOOT programs in math, computer science, chemistry, and physics.

Looking for summer camps in math and language arts? Be sure to check out the video-based summer camps offered at the Virtual Campus that are 2- to 4-weeks in duration. There are middle and high school competition math camps as well as Math Beasts camps that review key topics coupled with fun explorations covering areas such as graph theory (Math Beasts Camp 6), cryptography (Math Beasts Camp 7-8), and topology (Math Beasts Camp 8-9)!

Be sure to mark your calendars for the following events:
[list][*]April 3rd (Webinar), 4pm PT/7:00pm ET, Learning with AoPS: Perspectives from a Parent, Math Camp Instructor, and University Professor
[*]April 8th (Math Jam), 4:30pm PT/7:30pm ET, 2025 MATHCOUNTS State Discussion
April 9th (Webinar), 4:00pm PT/7:00pm ET, Learn about Video-based Summer Camps at the Virtual Campus
[*]April 10th (Math Jam), 4:30pm PT/7:30pm ET, 2025 MathILy and MathILy-Er Math Jam: Multibackwards Numbers
[*]April 22nd (Webinar), 4:00pm PT/7:00pm ET, Competitive Programming at AoPS (USACO).[/list]
Our full course list for upcoming classes is below:
All classes run 7:30pm-8:45pm ET/4:30pm - 5:45pm PT unless otherwise noted.

Introductory: Grades 5-10

Prealgebra 1 Self-Paced

Prealgebra 1
Sunday, Apr 13 - Aug 10
Tuesday, May 13 - Aug 26
Thursday, May 29 - Sep 11
Sunday, Jun 15 - Oct 12
Monday, Jun 30 - Oct 20
Wednesday, Jul 16 - Oct 29

Prealgebra 2 Self-Paced

Prealgebra 2
Sunday, Apr 13 - Aug 10
Wednesday, May 7 - Aug 20
Monday, Jun 2 - Sep 22
Sunday, Jun 29 - Oct 26
Friday, Jul 25 - Nov 21

Introduction to Algebra A Self-Paced

Introduction to Algebra A
Monday, Apr 7 - Jul 28
Sunday, May 11 - Sep 14 (1:00 - 2:30 pm ET/10:00 - 11:30 am PT)
Wednesday, May 14 - Aug 27
Friday, May 30 - Sep 26
Monday, Jun 2 - Sep 22
Sunday, Jun 15 - Oct 12
Thursday, Jun 26 - Oct 9
Tuesday, Jul 15 - Oct 28

Introduction to Counting & Probability Self-Paced

Introduction to Counting & Probability
Wednesday, Apr 16 - Jul 2
Thursday, May 15 - Jul 31
Sunday, Jun 1 - Aug 24
Thursday, Jun 12 - Aug 28
Wednesday, Jul 9 - Sep 24
Sunday, Jul 27 - Oct 19

Introduction to Number Theory
Thursday, Apr 17 - Jul 3
Friday, May 9 - Aug 1
Wednesday, May 21 - Aug 6
Monday, Jun 9 - Aug 25
Sunday, Jun 15 - Sep 14
Tuesday, Jul 15 - Sep 30

Introduction to Algebra B Self-Paced

Introduction to Algebra B
Wednesday, Apr 16 - Jul 30
Tuesday, May 6 - Aug 19
Wednesday, Jun 4 - Sep 17
Sunday, Jun 22 - Oct 19
Friday, Jul 18 - Nov 14

Introduction to Geometry
Wednesday, Apr 23 - Oct 1
Sunday, May 11 - Nov 9
Tuesday, May 20 - Oct 28
Monday, Jun 16 - Dec 8
Friday, Jun 20 - Jan 9
Sunday, Jun 29 - Jan 11
Monday, Jul 14 - Jan 19

Intermediate: Grades 8-12

Intermediate Algebra
Monday, Apr 21 - Oct 13
Sunday, Jun 1 - Nov 23
Tuesday, Jun 10 - Nov 18
Wednesday, Jun 25 - Dec 10
Sunday, Jul 13 - Jan 18
Thursday, Jul 24 - Jan 22

Intermediate Counting & Probability
Wednesday, May 21 - Sep 17
Sunday, Jun 22 - Nov 2

Intermediate Number Theory
Friday, Apr 11 - Jun 27
Sunday, Jun 1 - Aug 24
Wednesday, Jun 18 - Sep 3

Precalculus
Wednesday, Apr 9 - Sep 3
Friday, May 16 - Oct 24
Sunday, Jun 1 - Nov 9
Monday, Jun 30 - Dec 8

Advanced: Grades 9-12

Olympiad Geometry
Tuesday, Jun 10 - Aug 26

Calculus
Tuesday, May 27 - Nov 11
Wednesday, Jun 25 - Dec 17

Group Theory
Thursday, Jun 12 - Sep 11

Contest Preparation: Grades 6-12

MATHCOUNTS/AMC 8 Basics
Wednesday, Apr 16 - Jul 2
Friday, May 23 - Aug 15
Monday, Jun 2 - Aug 18
Thursday, Jun 12 - Aug 28
Sunday, Jun 22 - Sep 21
Tues & Thurs, Jul 8 - Aug 14 (meets twice a week!)

MATHCOUNTS/AMC 8 Advanced
Friday, Apr 11 - Jun 27
Sunday, May 11 - Aug 10
Tuesday, May 27 - Aug 12
Wednesday, Jun 11 - Aug 27
Sunday, Jun 22 - Sep 21
Tues & Thurs, Jul 8 - Aug 14 (meets twice a week!)

AMC 10 Problem Series
Friday, May 9 - Aug 1
Sunday, Jun 1 - Aug 24
Thursday, Jun 12 - Aug 28
Tuesday, Jun 17 - Sep 2
Sunday, Jun 22 - Sep 21 (1:00 - 2:30 pm ET/10:00 - 11:30 am PT)
Monday, Jun 23 - Sep 15
Tues & Thurs, Jul 8 - Aug 14 (meets twice a week!)

AMC 10 Final Fives
Sunday, May 11 - Jun 8
Tuesday, May 27 - Jun 17
Monday, Jun 30 - Jul 21

AMC 12 Problem Series
Tuesday, May 27 - Aug 12
Thursday, Jun 12 - Aug 28
Sunday, Jun 22 - Sep 21
Wednesday, Aug 6 - Oct 22

AMC 12 Final Fives
Sunday, May 18 - Jun 15

F=ma Problem Series
Wednesday, Jun 11 - Aug 27

WOOT Programs
Visit the pages linked for full schedule details for each of these programs!


MathWOOT Level 1
MathWOOT Level 2
ChemWOOT
CodeWOOT
PhysicsWOOT

Programming

Introduction to Programming with Python
Thursday, May 22 - Aug 7
Sunday, Jun 15 - Sep 14 (1:00 - 2:30 pm ET/10:00 - 11:30 am PT)
Tuesday, Jun 17 - Sep 2
Monday, Jun 30 - Sep 22

Intermediate Programming with Python
Sunday, Jun 1 - Aug 24
Monday, Jun 30 - Sep 22

USACO Bronze Problem Series
Tuesday, May 13 - Jul 29
Sunday, Jun 22 - Sep 1

Physics

Introduction to Physics
Wednesday, May 21 - Aug 6
Sunday, Jun 15 - Sep 14
Monday, Jun 23 - Sep 15

Physics 1: Mechanics
Thursday, May 22 - Oct 30
Monday, Jun 23 - Dec 15

Relativity
Sat & Sun, Apr 26 - Apr 27 (4:00 - 7:00 pm ET/1:00 - 4:00pm PT)
Mon, Tue, Wed & Thurs, Jun 23 - Jun 26 (meets every day of the week!)
0 replies
jlacosta
Apr 2, 2025
0 replies
k i Adding contests to the Contest Collections
dcouchman   1
N Apr 5, 2023 by v_Enhance
Want to help AoPS remain a valuable Olympiad resource? Help us add contests to AoPS's Contest Collections.

Find instructions and a list of contests to add here: https://artofproblemsolving.com/community/c40244h1064480_contests_to_add
1 reply
dcouchman
Sep 9, 2019
v_Enhance
Apr 5, 2023
k i Zero tolerance
ZetaX   49
N May 4, 2019 by NoDealsHere
Source: Use your common sense! (enough is enough)
Some users don't want to learn, some other simply ignore advises.
But please follow the following guideline:


To make it short: ALWAYS USE YOUR COMMON SENSE IF POSTING!
If you don't have common sense, don't post.


More specifically:

For new threads:


a) Good, meaningful title:
The title has to say what the problem is about in best way possible.
If that title occured already, it's definitely bad. And contest names aren't good either.
That's in fact a requirement for being able to search old problems.

Examples:
Bad titles:
- "Hard"/"Medium"/"Easy" (if you find it so cool how hard/easy it is, tell it in the post and use a title that tells us the problem)
- "Number Theory" (hey guy, guess why this forum's named that way¿ and is it the only such problem on earth¿)
- "Fibonacci" (there are millions of Fibonacci problems out there, all posted and named the same...)
- "Chinese TST 2003" (does this say anything about the problem¿)
Good titles:
- "On divisors of a³+2b³+4c³-6abc"
- "Number of solutions to x²+y²=6z²"
- "Fibonacci numbers are never squares"


b) Use search function:
Before posting a "new" problem spend at least two, better five, minutes to look if this problem was posted before. If it was, don't repost it. If you have anything important to say on topic, post it in one of the older threads.
If the thread is locked cause of this, use search function.

Update (by Amir Hossein). The best way to search for two keywords in AoPS is to input
[code]+"first keyword" +"second keyword"[/code]
so that any post containing both strings "first word" and "second form".


c) Good problem statement:
Some recent really bad post was:
[quote]$lim_{n\to 1}^{+\infty}\frac{1}{n}-lnn$[/quote]
It contains no question and no answer.
If you do this, too, you are on the best way to get your thread deleted. Write everything clearly, define where your variables come from (and define the "natural" numbers if used). Additionally read your post at least twice before submitting. After you sent it, read it again and use the Edit-Button if necessary to correct errors.


For answers to already existing threads:


d) Of any interest and with content:
Don't post things that are more trivial than completely obvious. For example, if the question is to solve $x^{3}+y^{3}=z^{3}$, do not answer with "$x=y=z=0$ is a solution" only. Either you post any kind of proof or at least something unexpected (like "$x=1337, y=481, z=42$ is the smallest solution). Someone that does not see that $x=y=z=0$ is a solution of the above without your post is completely wrong here, this is an IMO-level forum.
Similar, posting "I have solved this problem" but not posting anything else is not welcome; it even looks that you just want to show off what a genius you are.

e) Well written and checked answers:
Like c) for new threads, check your solutions at least twice for mistakes. And after sending, read it again and use the Edit-Button if necessary to correct errors.



To repeat it: ALWAYS USE YOUR COMMON SENSE IF POSTING!


Everything definitely out of range of common sense will be locked or deleted (exept for new users having less than about 42 posts, they are newbies and need/get some time to learn).

The above rules will be applied from next monday (5. march of 2007).
Feel free to discuss on this here.
49 replies
ZetaX
Feb 27, 2007
NoDealsHere
May 4, 2019
2024 IMO P1
EthanWYX2009   101
N 6 minutes ago by santhoshn
Source: 2024 IMO P1
Determine all real numbers $\alpha$ such that, for every positive integer $n,$ the integer
$$\lfloor\alpha\rfloor +\lfloor 2\alpha\rfloor +\cdots +\lfloor n\alpha\rfloor$$is a multiple of $n.$ (Note that $\lfloor z\rfloor$ denotes the greatest integer less than or equal to $z.$ For example, $\lfloor -\pi\rfloor =-4$ and $\lfloor 2\rfloor= \lfloor 2.9\rfloor =2.$)

Proposed by Santiago Rodríguez, Colombia
101 replies
EthanWYX2009
Jul 16, 2024
santhoshn
6 minutes ago
Telescopic Sum
P162008   1
N 14 minutes ago by Mathzeus1024
Compute the value of $\Omega = \sum_{r=1}^{\infty} \frac{14 - 9r - 90r^2 - 36r^3}{7^r  r(r + 1)(r + 2)(4r^2 - 1)}$
1 reply
P162008
Today at 1:06 AM
Mathzeus1024
14 minutes ago
Equal angles with midpoint of $AH$
Stuttgarden   2
N 18 minutes ago by HormigaCebolla
Source: Spain MO 2025 P4
Let $ABC$ be an acute triangle with circumcenter $O$ and orthocenter $H$, satisfying $AB<AC$. The tangent line at $A$ to the circumcicle of $ABC$ intersects $BC$ in $T$. Let $X$ be the midpoint of $AH$. Prove that $\angle ATX=\angle OTB$.
2 replies
Stuttgarden
Mar 31, 2025
HormigaCebolla
18 minutes ago
3 var inequalities
sqing   2
N 32 minutes ago by sqing
Source: Own
Let $ a,b> 0 $ and $ a+b\leq  2ab . $ Prove that
$$ \frac{   a + b }{ a^2(1+ b^2)} \leq\frac{1 }{\sqrt 2}-\frac{1 }{2}$$$$ \frac{   a +ab+ b }{ a^2(1+ b^2)} \leq \sqrt 2-1$$$$ \frac{   a +a^2b^2+ b }{ a^2(1+ b^2)} \leq\frac{\sqrt5 }{2}$$
2 replies
sqing
Yesterday at 1:13 PM
sqing
32 minutes ago
Flipping L's
MarkBcc168   12
N an hour ago by zRevenant
Source: IMO Shortlist 2023 C1
Let $m$ and $n$ be positive integers greater than $1$. In each unit square of an $m\times n$ grid lies a coin with its tail side up. A move consists of the following steps.
[list=1]
[*]select a $2\times 2$ square in the grid;
[*]flip the coins in the top-left and bottom-right unit squares;
[*]flip the coin in either the top-right or bottom-left unit square.
[/list]
Determine all pairs $(m,n)$ for which it is possible that every coin shows head-side up after a finite number of moves.

Thanasin Nampaisarn, Thailand
12 replies
MarkBcc168
Jul 17, 2024
zRevenant
an hour ago
\frac{1}{5-2a}
Havu   1
N 3 hours ago by Havu
Let $a,b,c \ge \frac{1}{2}$ and $a^2+b^2+c^2=3$. Find minimum:
\[P=\frac{1}{5-2a}+\frac{1}{5-2b}+\frac{1}{5-2c}.\]
1 reply
Havu
Yesterday at 9:56 AM
Havu
3 hours ago
<DPA+ <AQD =< QIP wanted, incircle circumcircle related
parmenides51   41
N 3 hours ago by Ilikeminecraft
Source: IMo 2019 SL G6
Let $I$ be the incentre of acute-angled triangle $ABC$. Let the incircle meet $BC, CA$, and $AB$ at $D, E$, and $F,$ respectively. Let line $EF$ intersect the circumcircle of the triangle at $P$ and $Q$, such that $F$ lies between $E$ and $P$. Prove that $\angle DPA + \angle AQD =\angle QIP$.

(Slovakia)
41 replies
parmenides51
Sep 22, 2020
Ilikeminecraft
3 hours ago
Pretty hard functional equation
vralex   5
N 3 hours ago by jasperE3
Source: National MO, 9th grade
Find all injective functions $ f:\mathbb{Z} \rightarrow \mathbb{Z} $ so that for every $n$ in $\mathbb{Z} ,  f (f (n))-f(n)-1=0$.
5 replies
vralex
Apr 29, 2020
jasperE3
3 hours ago
Parallelity and equal angles given, wanted an angle equality
BarisKoyuncu   5
N 4 hours ago by SleepyGirraffe
Source: 2022 Turkey JBMO TST P4
Given a convex quadrilateral $ABCD$ such that $m(\widehat{ABC})=m(\widehat{BCD})$. The lines $AD$ and $BC$ intersect at a point $P$ and the line passing through $P$ which is parallel to $AB$, intersects $BD$ at $T$. Prove that
$$m(\widehat{ACB})=m(\widehat{PCT})$$
5 replies
BarisKoyuncu
Mar 15, 2022
SleepyGirraffe
4 hours ago
Cyclic points and concurrency [1st Lemoine circle]
shobber   10
N 5 hours ago by Ilikeminecraft
Source: China TST 2005
Let $\omega$ be the circumcircle of acute triangle $ABC$. Two tangents of $\omega$ from $B$ and $C$ intersect at $P$, $AP$ and $BC$ intersect at $D$. Point $E$, $F$ are on $AC$ and $AB$ such that $DE \parallel BA$ and $DF \parallel CA$.
(1) Prove that $F,B,C,E$ are concyclic.

(2) Denote $A_{1}$ the centre of the circle passing through $F,B,C,E$. $B_{1}$, $C_{1}$ are difined similarly. Prove that $AA_{1}$, $BB_{1}$, $CC_{1}$ are concurrent.
10 replies
shobber
Jun 27, 2006
Ilikeminecraft
5 hours ago
Hard functional equation
Jessey   4
N 5 hours ago by jasperE3
Source: Belarus 2005
Find all functions $f:N -$> $N$ that satisfy $f(m-n+f(n)) = f(m)+f(n)$, for all $m, n$$N$.
4 replies
Jessey
Mar 11, 2020
jasperE3
5 hours ago
Vertices of a convex polygon if and only if m(S) = f(n)
orl   12
N 5 hours ago by Maximilian113
Source: IMO Shortlist 2000, C3
Let $ n \geq 4$ be a fixed positive integer. Given a set $ S = \{P_1, P_2, \ldots, P_n\}$ of $ n$ points in the plane such that no three are collinear and no four concyclic, let $ a_t,$ $ 1 \leq t \leq n,$ be the number of circles $ P_iP_jP_k$ that contain $ P_t$ in their interior, and let \[m(S)=a_1+a_2+\cdots + a_n.\]Prove that there exists a positive integer $ f(n),$ depending only on $ n,$ such that the points of $ S$ are the vertices of a convex polygon if and only if $ m(S) = f(n).$
12 replies
orl
Aug 10, 2008
Maximilian113
5 hours ago
Imo Shortlist Problem
Lopes   35
N 5 hours ago by Maximilian113
Source: IMO Shortlist 2000, Problem N4
Find all triplets of positive integers $ (a,m,n)$ such that $ a^m + 1 \mid (a + 1)^n$.
35 replies
Lopes
Feb 27, 2005
Maximilian113
5 hours ago
Inspired by Humberto_Filho
sqing   0
6 hours ago
Source: Own
Let $ a,b\geq 0 $ and $a + b \leq 2$. Prove that
$$\frac{a^2+1}{(( a+ b)^2+1)^2} \geq  \frac{1}{25} $$$$\frac{(a^2+1)(b^2+1)}{((a+b)^2+1)^2} \geq  \frac{4}{25} $$$$ \frac{a^2+1}{(( a+ 2b)^2+1)^2} \geq  \frac{1}{289} $$$$ \frac{a^2+1}{((2a+ b)^2+1)^2} \geq  \frac{5}{289} $$


0 replies
sqing
6 hours ago
0 replies
one cyclic formed by two cyclic
CrazyInMath   37
N Apr 19, 2025 by G81928128
Source: EGMO 2025/3
Let $ABC$ be an acute triangle. Points $B, D, E$, and $C$ lie on a line in this order and satisfy $BD = DE = EC$. Let $M$ and $N$ be the midpoints of $AD$ and $AE$, respectively. Suppose triangle $ADE$ is acute, and let $H$ be its orthocentre. Points $P$ and $Q$ lie on lines $BM$ and $CN$, respectively, such that $D, H, M,$ and $P$ are concyclic and pairwise different, and $E, H, N,$ and $Q$ are concyclic and pairwise different. Prove that $P, Q, N,$ and $M$ are concyclic.
37 replies
CrazyInMath
Apr 13, 2025
G81928128
Apr 19, 2025
one cyclic formed by two cyclic
G H J
G H BBookmark kLocked kLocked NReply
Source: EGMO 2025/3
The post below has been deleted. Click to close.
This post has been deleted. Click here to see post.
CrazyInMath
446 posts
#1 • 10 Y
Y by farhad.fritl, Davud29_09, ehuseyinyigit, Rounak_iitr, dangerousliri, cubres, MathLuis, Frd_19_Hsnzde, mariairam, Funcshun840
Let $ABC$ be an acute triangle. Points $B, D, E$, and $C$ lie on a line in this order and satisfy $BD = DE = EC$. Let $M$ and $N$ be the midpoints of $AD$ and $AE$, respectively. Suppose triangle $ADE$ is acute, and let $H$ be its orthocentre. Points $P$ and $Q$ lie on lines $BM$ and $CN$, respectively, such that $D, H, M,$ and $P$ are concyclic and pairwise different, and $E, H, N,$ and $Q$ are concyclic and pairwise different. Prove that $P, Q, N,$ and $M$ are concyclic.
Z K Y
The post below has been deleted. Click to close.
This post has been deleted. Click here to see post.
WLOGQED1729
44 posts
#2 • 14 Y
Y by ehuseyinyigit, EeEeRUT, maxd3, cubres, malong, Rounak_iitr, farhad.fritl, Frd_19_Hsnzde, Patrik, khina, Qingzhou_Xu, Assassino9931, zaidova, jrpartty
Beautiful Problem! Here's my solution:

Let $B'$ be the reflection of $B$ over $M$, and let $C'$ be the reflection of $C$ over $N$.
It is clear that $A$ is the midpoint of $B'C'$, and $B'C' \parallel MN \parallel BC$.

Claim: $\angle HB'C' = \angle HC'B' = 90^\circ - \angle DAE$

Proof:
Note that $H, A, B', E$ are concyclic and $H, A, C', D$ are also concyclic.
The rest follows from simple angle chasing.$\blacksquare$

Since $P, M, H, D$ are concyclic, we have:
\[
\angle HPB' = \angle HPM = \angle MDH = 90^\circ - \angle DAE
\]Similarly,
\[
\angle HQC' = 90^\circ - \angle DAE
\]
By the claim, we know:
\[
\angle HPB' = \angle HC'B' \quad \text{and} \quad \angle HQC' = \angle HB'C'
\]So, the points $B', C', P, Q, H$ lie on a circle.

Finally, notice that $MN \parallel B'C'$.
Applying Reim's Theorem yields that $M, N, P, Q$ are concyclic. $\blacksquare$
Attachments:
This post has been edited 3 times. Last edited by WLOGQED1729, Apr 13, 2025, 4:45 PM
Z K Y
The post below has been deleted. Click to close.
This post has been deleted. Click here to see post.
hectorleo123
342 posts
#3 • 5 Y
Y by maxd3, cubres, MathLuis, Rsantiaguito123, Gato_combinatorio
Easy for P3 :)
Let \( M' \) and \( N' \) be points on \( AM \) and \( AN \) such that \( \frac{AM'}{M'M} = \frac{AN'}{N'N} = 2 \).
By Menelaus on triangle \( ADE \) with transversal \( N'M \), we get that \( N' \), \( M \), and \( B \) are collinear.
Similarly, \( M' \), \( N \), and \( C \) are collinear.

Now let \( G' = MN' \cap N'M \).
We want to prove that \( G'M \cdot G'P = G'N \cdot G'Q \) if and only if \( G' \) lies on the radical axis of \( (DHM) \) and \( (EHN) \).

Let \( O \) be the circumcenter of triangle \( AMN \). It is known that \( M, O, N, K \) are concyclic,($K$ is the second point of intersection of $(DHM)$ and $(EHN)$)
Since \( O \) is the midpoint of arc \( MN \), we have \( \angle MKO = \angle OKN = 90^\circ - \angle DAE \).
Hence, \( O \) lies on the radical axis of \( (DHM) \) and \( (EHN) \).
Now it suffices to prove that \( H \), \( G' \), and \( O \) are collinear.

Note that \( H \) is the reflection of \( A \) over \( H' \),($H'$ is the orthocenter of triangle \( AMN \), )
and \( G' \) is such that \( \frac{AG}{GG'} = 5 \) (\( G \) is the centroid of triangle \( AMN \))
By Menelaus on triangle \( AH'G \), we are done.
This post has been edited 6 times. Last edited by hectorleo123, Apr 13, 2025, 3:17 PM
Z K Y
The post below has been deleted. Click to close.
This post has been deleted. Click here to see post.
quacksaysduck
45 posts
#4 • 3 Y
Y by mashumaro, ja., cubres
Solved with Click to reveal hidden text

Solution
This post has been edited 2 times. Last edited by quacksaysduck, Apr 13, 2025, 1:59 PM
Reason: e
Z K Y
The post below has been deleted. Click to close.
This post has been deleted. Click here to see post.
bin_sherlo
707 posts
#5 • 3 Y
Y by farhad.fritl, cubres, egxa
First, change $B,C$ and $D,E$. Let $DM\cap EN=T$, let $K$ be the midpoint of $BC$. Let $G$ be the centroid of $ABC$ and $AG\cap MN=F$. Notice that $A,T,K$ are collinear because $(\overline{AM},\overline{AN}),(\overline{AT},\overline{A BC_{\infty}}),(\overline{AD},\overline{AE})$ is an involution and $KD=KE,KB=KC$.
Also $-1=(B,BC_{\infty};D,C)=(A,F;T,G)$ hence a simple calculation gives $\frac{AT}{AK}=\frac{3}{5}$. Work on the complex plane. Note that $t=\frac{3a+b+c}{5}$. Let $O_B$ and $O_C$ be the circumcenters of $(BHM)$ and $(CHN)$. By the circumcenter formula we get $o_b=\frac{2b^2+2ab+3bc+ac}{2(b+c)}$ and $o_c=\frac{2c^2+2ac+3bc+ab}{2(b+c)}$.
\[\frac{t-h}{o_b-o_c}=\frac{\frac{2a+4b+4c}{5}}{\frac{(b-c)(a+2b+2c)}{2(b+c)}}=\frac{4}{5}.\frac{b+c}{b-c}\in i\mathbb{R}\]As desired.$\blacksquare$
Z K Y
The post below has been deleted. Click to close.
This post has been deleted. Click here to see post.
shanelin-sigma
155 posts
#6
Y by
Bary on $\triangle ADE$
Attachments:
Z K Y
The post below has been deleted. Click to close.
This post has been deleted. Click here to see post.
GrantStar
819 posts
#7 • 2 Y
Y by bin_sherlo, khina
What!!!!

Let $D'$ and $E'$ be the reflections of $E$ over $M$ and $D$ over $N$. By the length conditions, $E'C$ has midpoint $N$ and $D'B$ has midpoint $M$. Also, $D'E'$ has midpoint $A$, and clearly line $D'E'$ is parallel to $BC$. We show that $D'E'PQ$ is cyclic, which implies the result by Reim's theorem.

By orthocenter reflection, $DHAE'$ is cyclic. Thus
\[\measuredangle HPD' = \measuredangle HPM = \measuredangle  HDM = \measuredangle HDA = \measuredangle HE'A = \measuredangle HE'D'\]implying that $HPD'E'$ is cyclic. Similarly, $HQD'E'$ is cyclic, and we're done.
Z K Y
The post below has been deleted. Click to close.
This post has been deleted. Click here to see post.
EeEeRUT
64 posts
#8 • 1 Y
Y by acuri
Let the circumcenter of $(AMN)$ be $O$ and let $(HDM)$ intersect $(HEN)$ at $T \neq H$.
By radical axis, we are left to show that $BM, CN$ and $HT$ concurrent. Suppose $BM, CN$ meets at $I$.
Notice that $$\angle MON = 2\angle MAN = 180^{\circ} -\angle ADH - \angle AEH = 180^{\circ} - \angle MTN$$Hence, $M,N,O,T$ are concyclic. And since, $HT$ bisects $\angle MTN$( byAngle chasing), $M, T, O$ are collinear.
By Ceva, $I$ lies on median of $\triangle ABC$.
Let the midpoint of $MN$ be $Z$ and $K$ be midpoint of $BC$. By Menelaus, $$AI = 4IZ$$.
It is known that $$\frac{AH}{OZ} = 4$$This is why
Let $AZ$ intersect $OH$ at $I_1$, it follows that $$\frac{AI_1}{I_1Z}  = 4$$So, $$I_1 = I$$Hence, we are done $\blacksquare$.
[asy] /* Geogebra to Asymptote conversion, documentation at artofproblemsolving.com/Wiki go to User:Azjps/geogebra */
import graph; size(10cm); 
real labelscalefactor = 0.5; /* changes label-to-point distance */
pen dps = linewidth(0.7) + fontsize(10); defaultpen(dps); /* default pen style */ 
pen dotstyle = black; /* point style */ 
real xmin = -6.642733543503411, xmax = -0.4321934724055566, ymin = -2.7242230556319087, ymax = 6.6091309495207025;  /* image dimensions */
pen wwqqcc = rgb(0.4,0,0.8); pen fuqqzz = rgb(0.9568627450980393,0,0.6); pen qqwuqq = rgb(0,0.39215686274509803,0); 
 /* draw figures */
draw((-4.315243008383876,4.92491669295181)--(-6.145299148418266,2.1563307052402014), linewidth(0.8)); 
draw((-6.145299148418266,2.1563307052402014)--(-1.407818016280837,2.3107795394165724), linewidth(0.8)); 
draw((-1.407818016280837,2.3107795394165724)--(-4.315243008383876,4.92491669295181), linewidth(0.8)); 
draw((-4.440690889711499,3.5663651714587346)--(-3.651110701021928,3.592106643821463), linewidth(0.8)); 
draw((-4.099769237970145,3.848372064702441)--(-4.232088162837603,2.3742693285864003), linewidth(0.8) + linetype("4 4") + wwqqcc); 
draw((-4.066689506753282,4.216897748731452)--(-4.099769237970145,3.848372064702441), linewidth(0.8) + linetype("4 4") + wwqqcc); 
draw((-4.066689506753282,4.216897748731452)--(-4.045900795366713,3.579235907640099), linewidth(0.8) + blue); 
draw(circle((-4.052310876932325,3.7758553294343264), 0.44127673926345784), linewidth(0.8) + linetype("4 4")); 
draw(circle((-4.705415552255749,2.905742043450558), 0.7116895841716911), linewidth(0.8) + fuqqzz); 
draw(circle((-3.564604857693222,2.8033401952730443), 0.7934959176238369), linewidth(0.8) + fuqqzz); 
draw((-6.145299148418266,2.1563307052402014)--(-4.099769237970145,3.848372064702441), linewidth(0.8) + linetype("4 4")); 
draw((-4.045900795366713,3.579235907640099)--(-3.7765585823495513,2.233555122328387), linewidth(0.8)); 
draw((-4.099769237970145,3.848372064702441)--(-1.407818016280837,2.3107795394165724), linewidth(0.8) + linetype("4 4")); 
draw((-4.315243008383876,4.92491669295181)--(-4.566138771039123,2.2078136499656584), linewidth(0.8)); 
draw((-4.315243008383876,4.92491669295181)--(-2.98697839365998,2.2592965946911154), linewidth(0.8)); 
draw((-4.315243008383876,4.92491669295181)--(-4.232088162837603,2.3742693285864003), linewidth(0.8) + qqwuqq); 
draw((-4.099769237970145,3.848372064702441)--(-4.315243008383876,4.92491669295181), linewidth(0.8) + qqwuqq); 
draw((-4.099769237970145,3.848372064702441)--(-4.045900795366713,3.579235907640099), linewidth(0.8) + blue); 
 /* dots and labels */
dot((-4.315243008383876,4.92491669295181),dotstyle); 
label("$A$", (-4.26845927903474,5.041876016324649), NE * labelscalefactor); 
dot((-6.145299148418266,2.1563307052402014),dotstyle); 
label("$B$", (-6.093024723651059,2.269940052388348), NE * labelscalefactor); 
dot((-4.566138771039123,2.2078136499656584),dotstyle); 
label("$D$", (-4.5140738581177065,2.328419714074768), NE * labelscalefactor); 
dot((-2.98697839365998,2.2592965946911154),dotstyle); 
label("$E$", (-2.935122992584353,2.3752034434239038), NE * labelscalefactor); 
dot((-1.407818016280837,2.3107795394165724),dotstyle); 
label("$C$", (-1.3561721270510003,2.4219871727730395), NE * labelscalefactor); 
dot((-4.440690889711499,3.5663651714587346),linewidth(4pt) + dotstyle); 
label("$M$", (-4.3971145347448655,3.661756000525141), NE * labelscalefactor); 
dot((-3.651110701021928,3.592106643821463),linewidth(4pt) + dotstyle); 
label("$N$", (-3.6017911358095467,3.685147865199709), NE * labelscalefactor); 
dot((-4.232088162837603,2.3742693285864003),linewidth(4pt) + dotstyle); 
label("$H$", (-4.186587752673751,2.4687709021221758), NE * labelscalefactor); 
dot((-4.045900795366713,3.579235907640099),linewidth(4pt) + dotstyle); 
label("$Z$", (-3.999452835277206,3.6734519328624247), NE * labelscalefactor); 
dot((-4.099769237970145,3.848372064702441),linewidth(4pt) + dotstyle); 
label("$I$", (-4.057932496963627,3.9424583766199564), NE * labelscalefactor); 
dot((-4.066689506753282,4.216897748731452),linewidth(4pt) + dotstyle); 
label("$O$", (-4.022844699951774,4.30503227907576), NE * labelscalefactor); 
dot((-3.7765585823495513,2.233555122328387),linewidth(4pt) + dotstyle); 
label("$K$", (-3.7304463915196715,2.328419714074768), NE * labelscalefactor); 
dot((-4.145004800767755,3.344423994110871),linewidth(4pt) + dotstyle); 
label("$T$", (-4.093020293975479,3.4395332861167454), NE * labelscalefactor); 
clip((xmin,ymin)--(xmin,ymax)--(xmax,ymax)--(xmax,ymin)--cycle); 
 /* end of picture */
[/asy]
This post has been edited 1 time. Last edited by EeEeRUT, Apr 14, 2025, 2:17 AM
Reason: Angle chase
Z K Y
The post below has been deleted. Click to close.
This post has been deleted. Click here to see post.
MathLuis
1501 posts
#9 • 1 Y
Y by hectorleo123
Absolute gem of a problem, and the perfect finale for day 1.
Let $T$ the E-queue point of $\triangle ADE$ then let $A'$ a point such that $ADA'E$ is a parallelogram and also let $A'D \cap HE=S$ and $A'E \cap DH=R$, also let $K,L$ reflections of $A'$ over $E,D$ respectively then from parallelogram's spam and midbases checking and projective whatever you like you can easly check that $K,L,A$ are colinear on a line parallel to $BC$ but also $BM \cap DN=K$ and $CN \cap EM=L$, however from taking homothety with scale factor 2 from $A'$ we can see $(SHR)$ is the image of the NPC of $\triangle A'SR$ and thus $LSHRK$ is cyclic, now to finish just notice that if you let $H'$ reflection of $H$ over $M$ then it lies on $(ADE)$ and in fact $EH'$ is diameter so $H',M,H,T$ are colinear and by PoP $H'M \cdot MT=AM \cdot MD$ and thus $MH \cdot MT=DM^2$ which gives that $(DHT)$ is tangent to $AD$ and now let $ET \cap A'D'=S'$ then since $\angle HTS'=90=\angle HDS'$ we have $HDS'T$ cyclic but then $\angle DHS'=\angle DTS'=\angle DAE=\angle EA'D=\angle SHD$ and therefore $S,S'$ are reflections over $HD$ and thus $SD=DS'$ and thus $BSES'$ is a parallelogram which gives that $\angle BSD=\angle  DS'T=\angle ADT=\angle AH'T=\angle MHD=\angle BPD$ and thus $BPSD$ is cyclic and by Reim's it means $LPSK$ cyclic and repeating the same process for $Q$ and joining all the results gives $LPSHQRK$ cyclic and by Reim's this gives $PMNQ$ cyclic as desired thus we are done :cool:.
This post has been edited 2 times. Last edited by MathLuis, Apr 13, 2025, 3:41 PM
Z K Y
The post below has been deleted. Click to close.
This post has been deleted. Click here to see post.
mariairam
7 posts
#10
Y by
Let $\{S\}=(DMH)\cap(ENH)$. We prove that $SH, BM,CN$ concur, and the conclusion follows.
We prove that the intersection point is in fact the point that divides the $A$-median in a $\frac{2}{3}$ ratio.
Let $\{P'\}=BM\cap AC$, $\{Q'\}=CN\cap AB$, $\{T\}=BP'\cap CQ'$ and $\{R\}=AT\cap BC$.
$\boldsymbol{Claim:}$ $\frac{AT}{TR}=\frac{2}{3}$.
$\boldsymbol{Proof:}$ Applying Menelaus' Theorem in $\triangle ADC$ we get that $\frac{AP'}{P'C}=\frac{1}{3}$, and similarly $\frac{AQ'}{Q'B}=\frac{1}{3}$ as well. Then, by Ceva, $BR=CR$, and applying Menelaus' Theorem again, in $\triangle ARC$, the claim follows.

Now, let $\{T'\}=SH\cap AR$, $O$ be the center of $(ADE)$, $O'$ be the center of $(AMN)$ and $\{L\}=OR\cap SH$.
$\boldsymbol{Claim:}S,H,O' $ are collinear.
$\boldsymbol{Proof:}$ Since $D,S,H,M$ and $E,S,H,N$ are concyclic, we have that $\angle MSH=\angle NSH=90\textdegree - \angle DAE$, which quickly yields that $M,S,N,O'$ are concyclic. Since $O'M=O'N$, $SO'$ is the bisector of $\angle SMN$ -- but so is $SH$. Therefore, $S,H,O'$ are collinear.
All that's left for the problem's conclusion to be obtained is to find that $T=T'$, i.e. $\frac{AT'}{T'R}=\frac{2}{3}$.
By homothety, $O'$ is the midpoint of $AO$, and since $AH\parallel OL$, then $AHOL$ is a parallelogram and $OL=AH$. By Menelaus' Theorem in $\triangle ARO$, $\frac{AT'}{T'R}=\frac{OL}{LR}=\frac{AH}{AH+\frac{AH}{2}}=\frac{2}{3}$.
So $T=T'$, meaning that $SH, BM$ and $CN$ concur in $T$, and by power of point we have that $P,M,N,Q$ are concyclic.
Attachments:
Z K Y
The post below has been deleted. Click to close.
This post has been deleted. Click here to see post.
InterLoop
274 posts
#11
Y by
solution
Z K Y
The post below has been deleted. Click to close.
This post has been deleted. Click here to see post.
Pitchu-25
54 posts
#12
Y by
Let $X$ and $Y$ be points on the line through $A$ parallel to $BC$ with $AX=AY=DE$ and such that $X$ and $C$ lie on the same halfplane determined by line $AD$.
The point of the problem is to erase $P$ and $Q$ from the picture completely and try to prove that $Z:=(BM)\cap (CN)=(BX)\cap (CY)$ lies on the radical axis of circles $(DHM)$ and $(EHN)$.
Let $T$ be the point where circles $(DHM)$ and $(EHN)$ meet a second time and let $O'$ denote the circumcenter of $AMN$. We get $\angle MTH=\angle NTH=90-\angle BAC$, so that $T$ lies on both circle $(MNO')$ and line $(O'H)$. Therefore, it remains to show that points $H,Z$ and $O'$ are collinear.

Let $O$ denote the circumcenter of triangle $ADE$, so that $O'$ is the midpoint of $AO$. Let $G$ denote the centroid of triangle $ADE$, let $R$ denote the midpoint of $MN$ and let $W=(HR)\cap (AO)$.

Claim : Points $A,W,O'$ and $O$ are harmonic.
Proof : It suffices to show that $\frac{WA}{WO'}=4$. This follows from the fact that, if $L$ is the midpoint of $BC$, then $O'R=\frac{1}{2}OL=\frac{1}{4}AH$. $\square$

Furthermore, we have $A,R,Z$ and $G$ harmonic due to a complete quadrilateral. Therefore, by Prism Lemma, $(O'Z), (WR)$ and $(OG)$ must concur at a point, which turns out to be $H$ since $H\in (OG)$ and $H\in (WR)$.
It follows that $Z$ lies on line $(O'H)$, as needed.
$\blacksquare$
This post has been edited 1 time. Last edited by Pitchu-25, Apr 13, 2025, 6:15 PM
Z K Y
The post below has been deleted. Click to close.
This post has been deleted. Click here to see post.
cj13609517288
1891 posts
#13
Y by
I bashed this on paper, so this is a highly condensed summary. Let $X=BM\cap CN$, we want to show that it lies on the radax of $(DMH)$ and $(ENH)$. Now just bary wrt $ADE$. $X=(3:1:1)$ and set
\[A=\frac{S_{ABC}(a^2S_A+b^2S_B+c^2S_C)}{S_{AB}+S_{BC}+S_{CA}}.\]This is equal to $2S_{ABC}$ after some manipulation, and indeed it turns out that the radax will require $A=2S_{ABC}$. $\blacksquare$
This post has been edited 1 time. Last edited by cj13609517288, Apr 13, 2025, 6:57 PM
Z K Y
The post below has been deleted. Click to close.
This post has been deleted. Click here to see post.
HoRI_DA_GRe8
597 posts
#14
Y by
Quite Nice .
A wise man (Prabh2005) from my old days in Aops once wrote:
I am a simple man , whenever I see midpoints , I reflect .
It suffices to prove that $BM \cap CN =K$ lies on the radical axes of the 2 circles.Let $M$ be the midpoint of $DE$ (also $BC$) .

Claim : $A,K,M$ are collinear.
Proof : Reflect $B$ over $M$ ($B'$) and $C$ over $N$($C'$). It's easy to see that $B'$ is also the reflection $D$ over $M$ and $C'$ is the reflection of $C$ over $N$.Clearly $B'-K-B$ and $C'-K-C$ .Note that $B'-A-C'$ and $B'A=AC'$.So $A$ is the midpoint of $B'C'$ and $M$ is the midpoint of $BC$ and $K=BB' \cap CC'$, this clearly proves our claim that $A-K-M$ $\square$

Now note that $B'C' : BC=2 : 3$ which implies $AK : KM = 2:3$ as well. A bit of manipulations give that if $G$ is the centroid $AK:KG=3:2$. Let the circles meet again at $J$.Note that $HJ$ bisects $\angle MJN$ and also $\angle MJN=180-2\angle BAC$.Let $O'$ be the circumcentre of $\triangle AMN$. Clearly $O',M,J,N$ are concyclic and $O'J$ bisects $\angle MJN$ by the well known Incentre-Excentre Lemma or fact 5 or whatever the American Kids call it.So we have $O'-J-H$.

Final Claim : $H,J,K,O'$ are collinear.
Proof : Let $O$ be the circumcentre of $\triangle ABC$, clearly $O'$ is the midpoint of $AO$ and by Euler line ratios we have $OH:HG=3:2$.Now by Applying menelaus on $\triangle AGO$ we get that $K$ lies on $HO'$ $\square$
Now from the above claim we get that $K$ lies on the radical axis of the circles, The End $\blacksquare$
This post has been edited 1 time. Last edited by HoRI_DA_GRe8, Apr 13, 2025, 7:22 PM
Z K Y
The post below has been deleted. Click to close.
This post has been deleted. Click here to see post.
atdaotlohbh
185 posts
#15
Y by
It is easy to verify that $BM$ and $CN$ intersect on the median of triangle $ADE$ and cut it in the ratio $2 : 3$, say at $X$. We aim to prove that $X$ lies on the radical axis of $(DMH)$ and $(ENH)$. Let $Y$ be the second intersection point of this circles, and let $O$ be the circumcenter of triangle $AMN$. Then $\angle  MYN=\angle MYH+\angle NYH=\angle MDH+\angle HEN=180^{\circ}-2\angle DAE=180^{\circ}-\angle MON$, thus $M,O,N$ and $Y$ are concyclic. Also, $\angle MYH=\angle MDH=90^{\circ}-\angle MAN=\angle MNO = \angle MYO$, which means $O,H$ and $Y$ are collinear. Now our goal is to prove that $HO$ cuts the median in the ratio $2 : 3$. Let $O'$ be the circumcenter of $ADE$, from homothety it the reflection of $A$ in $O$. Let $K$ be the midpoint of $DE$. Let $O'K$ intersect $HO$ at $F$, and let $HO$ intersect $AK$ at $L$.Then $\frac{LK}{AL}=\frac{KF}{AH}=\frac{O'K+AH}{AH}=1+\frac{O'K}{AH}=1+\frac{1}{2}=\frac{3}{2}$, which is the desired ratio.
Z K Y
The post below has been deleted. Click to close.
This post has been deleted. Click here to see post.
ThatApollo777
73 posts
#16
Y by
We use barycentric coordinates with reference triangle $ADE$.
$$A=(1,0,0)$$$$D=(0,1,0)$$$$E=(0,0,1)$$$$B=(0,2,-1)$$$$C=(0,-1,2)$$$$M=(1/2, 1/2, 0)$$$$N=(1/2,0,1/2)$$$$BM : 2z + y - x = 0$$$$CN : 2y + z -x =0$$$$BM \cap CN = T = (3:1:1)$$Suffices to show $T$ has equal powers in $(DMH)$ and $(CNH)$.
$$(DMH) : -a^2yz - b^2 xz - c^2 xy + (x+y+z)(px + qz)=0$$Subbing coordinates of $M$ and $H$ and using conway's notation we get: $$p = \frac{c^2}{2}$$Dear reader, the author of this post had used $H = (S_A: S_B: S_C)$ in a previous edit and was bashing his head against a wall when the expressions won't cancel, please send help.
$$q = \frac{\frac{S_{ABC}\sum_{cyc}a^2S_{A}}{\sum_{cyc}S_{BC}} - \frac{c^2S_{BC}}{2}}{S_{AB}} = \frac{\frac{S_{ABC}(2S^2)}{S^2} - \frac{S_{ABC}+S_{BBC}}{2}}{S_{AB}} = 1.5S_C - \frac{S_{BC}}{S_A}$$$$pow(T, (DMH)) = \frac{1}{25}(-a^2-3b^2-3c^2+5(1.5c^2 + 1.5S_C - \frac{S_{BC}}{S_A}))$$This is clearly symmetric in $b$ and $c$ ($S_B + b^2 = S_C + c^2$) so we are done.
This post has been edited 3 times. Last edited by ThatApollo777, Apr 13, 2025, 9:28 PM
Reason: typo
Z K Y
The post below has been deleted. Click to close.
This post has been deleted. Click here to see post.
sbealing
308 posts
#17
Y by
Denote circles $DHM$ and $EHN$ by $\omega_{D}$ and $\omega_{E}$, respectively. Let $L$ and $G$ lie on $\omega_{D}$ and $\omega_{E}$, respectively such that $H$ lies on line $LG$ and $LG \parallel BC$. Let lines $LM$ and $GN$ intersect at $S$. Angle chasing and using $LG \parallel DE \parallel MN$ we get
$$\angle NMS=\angle GLS=\angle HLM=\angle HDM=\angle HDA=90^{\circ}-\angle DAE=90^{\circ}-\angle MAN.$$Similarly, $\angle SNM=90^{\circ}-\angle MAN$ which, combined, are enough to show that $S$ is the circumcentre of triangle $AMN$. Since $SM=SN$ and $MN \parallel LG$, we have $SM \cdot SL=SN \cdot SG$ so $S$ lies on the radical axis of $\omega_{D}$ and $\omega_{E}$.

Let $BM$ and $CN$ intersect at $K$. With respect to reference triangle $ADE$ with ${a},{d},{e}$ all unit vectors, we have (by Menelaus or areal coordinates), ${k}=\frac{3{a}+{d}+{e}}{5}$. We also have ${h}={a}+{d}+{e}$ and ${s}=\frac{{a}}{2}$ (since it is the midpoint of $A$ and the circumcentre of $ADE$). Thus, ${k}=\frac{4{s}+{h}}{5}$ so $H,K,S$ are collinear (with $SK:KH=1:4$).

Clearly $H$ lies on the radical axis of $\omega_{D}$ and $\omega_{E}$ so, as $S$ lies on this radical axis, so does $K$. Applying power of a point, we get
$$KM \cdot KP=\mathrm{Pow}_{\omega_{D}}{(K)}=\mathrm{Pow}_{\omega_{E}}{(K)}=KN \cdot KQ$$so $MPQN$ is cyclic as required.

Remark: The post here has a proof for $S$ being the circumcentre of triangle $AMN$.

https://i.ibb.co/4Bg6Cf2/EGMO-2025-P3.png
This post has been edited 2 times. Last edited by sbealing, Apr 14, 2025, 7:38 AM
Z K Y
The post below has been deleted. Click to close.
This post has been deleted. Click here to see post.
TestX01
340 posts
#18
Y by
hi orz

Let $O$ be circumcentre of $(AMN)$. Well known by APMO 18/1 $O$ lies on the radax of $(DHM)$ and $(EHN)$. Now, by Menelaus, if $G$ is the intersection of $BM$ and $CN$ then $G$ is on the median with a ratio $2:3$. Meanwhile, by radax theorem we want $G$ to be on radax of $(DHM)$ and $(EHN)$. Hence, RTP $H,O,G$ collinear. Now in complex, $O=\frac{a}{2}$, $H$ is $a+b+c$, and $G$ is $\frac{2}{5}\left(\frac{b+c}{2}-a\right)+a=\frac{b+c+3a}{5}$. Now, these points are collinear as $OH$ is $\frac{a}{2}+b+c$, meanwhile $OG$ is $\frac{b+c+\frac{a}{2}}{5}$ note the obvious scalar multiple.
This post has been edited 1 time. Last edited by TestX01, Apr 13, 2025, 10:43 PM
Z K Y
The post below has been deleted. Click to close.
This post has been deleted. Click here to see post.
MathSaiyan
75 posts
#19
Y by
lpop works nicely.

Let $K = BM\cap CN$. Define
\[
f(X) = pow_{(DNE)}(X) - pow_{(DMH)}(X).
\]We are done if $f(K) = 0$. The idea is to compute $f$ at $A,M,N$, then use lpop to finish. We work with $\triangle ADE$ as the reference, letting $\alpha,\delta,\epsilon$ be its angles, and $a,d,e$ be its sides, as usual. Clearly $f(A) = d^2/2 - e^2/2$.

The trick to compute $f(M)$ (and similarly for $N$) easily is to define $U$ to be the second intersection of $MN$ with $(EHN)$.
$MN$ is just $a/2$. On the other hand, notice that $\angle MUH = \angle NEH = 90-\alpha$. Hence, if we let $P$ be the foot of the perpendicular from $H$ to $MN$, we can compute
\[
MU = MP+PU = \frac{e}{2}\cos \delta + PH\tan\alpha.
\]As $PH\tan\alpha$ is symmetric in our reference, we don't even need to compute it (even though it is doable). So, now,
\[
f(M) = MN\cdot MU = \frac{ae}{4}\cos\delta + \frac{a}{2}PH\tan\alpha.
\]And similarly, if $V$ is the second intersection of line $MN$ with $(DHM)$,
\[
f(N) = -MN\cdot NV = -\frac{ad}{4}\cos\epsilon - \frac{a}{2}PH\tan\alpha.
\]Now we claim that $K = \frac{2}{5}M + \frac{2}{5}N + \frac{1}{5}A$. This is routine bary-like lengthchasing. For instance, if we let $R = KC\cap AM$, we can see that $AR = 2AM$ by using Manelaus' on $ENC$. After doing similarly from the other side, the claim follows.

So we're done, as then we just need to see that
\[
0 = 5f(k) = 2f(M)+2f(N)+f(A) = \frac{ae}{2}\cos\delta + aPH\tan\alpha -\frac{ad}{2}\cos\epsilon - aPH\tan\alpha + \frac{d^2 - e^2}{2}
\]But everything cancels nicely and the equality reduces to
\[
ae\cos\delta-ad\cos\epsilon + d^2 - e^2 = 0.
\]This follows directly from using cosine law to plug $\cos\delta = \frac{a^2+e^2-d^2}{2ae}$ and $\cos\epsilon = \frac{a^2+d^2-e^2}{2ad}$.
Attachments:
Z K Y
The post below has been deleted. Click to close.
This post has been deleted. Click here to see post.
TestX01
340 posts
#20
Y by
@above,
nice, I tried that but just didn't know how to express $f(M)$ in terms of trig.

Another linpop approach is to actually linpop on $BM$. Note that we know $MK$ and $BM$'s ratios etc because of homothety reasons.
Z K Y
The post below has been deleted. Click to close.
This post has been deleted. Click here to see post.
pingupignu
49 posts
#21
Y by
Here's an alternative way to linpop bash using $D$, $E$ by introducing Ptolemy's theorem.
Attachments:
This post has been edited 1 time. Last edited by pingupignu, Apr 14, 2025, 1:01 AM
Z K Y
The post below has been deleted. Click to close.
This post has been deleted. Click here to see post.
MarkBcc168
1595 posts
#22 • 1 Y
Y by GeoKing
Let $S$ be the circumcenter of $\triangle AMN$, and let $\odot(DHM)$ and $\odot(EHN)$ meet again at $X\neq H$.

Claim. $X, H, S$ are collinear.

Proof. Notice that $\angle SMN = \angle 90^\circ - \angle DAE = \angle ADH = \angle MXH$, and similarly, $\angle SNM = \angle NXH$. Hence, $\angle SMN = \angle SNM = \angle MXH = \angle NXH$. Thus, $MSNX$ is cyclic, which immediately yields the desired collinearity. $\blacksquare$.

Now, we let
  • $T$ be the midpoint of $DE$.
  • $G$ and $O$ be the centroid and circumcenter of $\triangle ADE$.
  • $HS$ intersects $AT$ at point $K$.
Then, by Menelaus theorem on $\triangle AGO$, we find that
$$\frac{AK}{KG} \cdot \frac{GH}{HO} \cdot \frac{OS}{SA} = 1 
\implies \frac{AK}{KG} = \frac 32.$$Hence, $AK = \tfrac 35 AG = \tfrac 25 AT$, which implies that $AK : KT  = 2:3$. Finally, Menelaus's theorem on $\triangle ABM$ gives that $B, K, M$ are collinear, and similarly, $C, K, N$ are collinear. Power of point at $K$ yields $$KP\cdot KM = KH\cdot KS = KQ \cdot KN,$$which gives the result.
This post has been edited 1 time. Last edited by MarkBcc168, Apr 14, 2025, 3:59 AM
Z K Y
The post below has been deleted. Click to close.
This post has been deleted. Click here to see post.
ItzsleepyXD
108 posts
#23
Y by
Let $O,G$ be circumcenter and centroid of $\triangle ADE$ respectively, $A'$ be point that $ADA'E$ is parallelogram.
$X$ is on $(DHM)$ such that $XH // DE$ , $Y$ is on $(EHN)$ such that $YH // DE$ .
$K = XM \cap YN , Z = KH \cap AA'$
claim 1 $K \in rad((DHM),(EHN))$ .
$\angle KXH = \angle MDH = 90^{\circ} - \angle DAE = \angle NEH = \angle HYK$
so $KX = KY$ and $KM = KN$ implies that $K \in rad((DHM),(EHN))$

also known that $\angle MKN = 2 \cdot \angle DAE$ so $K$ is circumcenter of $(AMN)$ .
and $K$ is midpoint of $AO$ .
so $(A,O;K, \infty_{AO}) = -1$ and $HA' // AO$ implies that $(A,G;Z,A') = -1$.
so $\frac{AZ}{ZG} = \frac{3}{2}$ it is easy to see by menelos that $Z,M,B$ and $Z,N,C$ collinear.
done. $\square$
This post has been edited 1 time. Last edited by ItzsleepyXD, Apr 14, 2025, 7:52 AM
Z K Y
The post below has been deleted. Click to close.
This post has been deleted. Click here to see post.
Funcshun840
22 posts
#24 • 1 Y
Y by Nicio9
Linpop nearly trivialises this problem :blush:

WLOG $AB < AC$. We first define some points: $R=BM \cap CN$, $Z$ the midpoint of $MN$, $Y=AH \cap MN$, $S=(DMH) \cap MN$, $T= (CNH) \cap MN$.

Claim: $R$ lies on $AZ$ such that $\frac{AR}{RZ} = \frac{4}{1}$
Proof: Call $MR \cap AE = K$ and $NR \cap AD = L$. By Menelaus on $ADE$, we see that $\frac{AK}{KC} = \frac{AL}{LD} = \frac{1}{3}$, implying that $\frac{AK}{KN} = \frac{AL}{LM} = 2$. This also implies by Ceva that $R$ lies on the median with $\frac{AR}{RZ} = 4$.

Claim: $HS = HT$, and thus $Y$ is the midpoint of $ST$.
Proof: We have $\angle HST = \angle HDA = \angle HEA = \angle HTS$.

Now define the difference of powers functions $f(P) = Pow(P, (ENH)) - Pow (P, (DMH))$. By the radical axis theorem, it suffices to show that $HR$ is the radical axis of the two circles, but since $\frac{AR}{RZ} = \frac{4}{1}$, it suffices by linpop to show that $f(A) = - 4 f(Z)$.

Call $a,b,c$ the lengths of sides $MN, AN, AM$. Then clearly $f(A) = 2(b^2 - c^2)$.
For $Z$, we have $-f(Z) = ZM \cdot ZS - ZN \cdot ZT= a \cdot (ZS - ZT) = \frac{a}{2} 2 YZ =\frac{1}{2} a \cdot (NY-MY) = a \cdot \frac{NY^2 - MY^2}{2a} = \frac{b^2 - c^2}{2}$.

Hence $f(A) = - 4 f(Z)$, so $R$ does indeed lie on the radical axis.
Attachments:
This post has been edited 3 times. Last edited by Funcshun840, Apr 14, 2025, 1:35 PM
Reason: sign error
Z K Y
The post below has been deleted. Click to close.
This post has been deleted. Click here to see post.
SimplisticFormulas
97 posts
#26 • 1 Y
Y by L13832
sol
Z K Y
The post below has been deleted. Click to close.
This post has been deleted. Click here to see post.
ThatApollo777
73 posts
#27
Y by
(ignore this post, i cant read)
This post has been edited 1 time. Last edited by ThatApollo777, Apr 17, 2025, 11:18 AM
Reason: skillissue
Z K Y
The post below has been deleted. Click to close.
This post has been deleted. Click here to see post.
L13832
262 posts
#28 • 1 Y
Y by alexanderhamilton124
Let $O$ be the circumcenter of $(AMN)$, $R=PM\cap QN$ and $G=(MDH)\cap (NEH)$. By radax the problem is equivalent to proving $PM,QN,GH$ are concurrent at $R$.

Note that $\overline{O-G-H}$ are collinear because
\begin{align*}
&\angle ONM = \frac{\angle 180^{\circ} - 2\angle MAN}{2}
=\angle ADH= 180^{\circ}-\angle MGH\\
\implies &\angle NGH=180^{\circ}-\angle OMN=180^{\circ}-\angle OGN.
\end{align*}By letting reflections of $B$ and $C$ over $M$ and $N$ be $B', C'$ we see that $\overline{C'-A-B'}$ are collinear and we get that $AB'=AC'$ as $C'ADB$ and $B'AEC$ are parallelograms. Since $R=BB'\cap CC'$ we get that $AR$ intersects $BC$ at the midpoint of $DE$ or $BC$, $I$. Now all we need to prove is $R\in \overline{O-G-H}$.

Note that $\frac{B'C'}{BC}=\frac 23$ so $\frac{AR}{RI}=\frac{2}{3}$, motivated by this we consider the centroid and circumcenter of $(AMN)$ to be $J,P$ so that $\frac{AJ}{AI}=\frac 23$, $AO=OP$ and $\frac{JH}{HP}=\frac 23$.
Defining $OH\cap AI=R'$ and by applying menelaus on $\triangle APJ$ we get that $\frac{AR'}{R'J} \cdot \frac{JH}{HP} \cdot \frac{PO}{OA} = 1 
\implies \frac{R'J}{AR'} = \frac 23$, so $R\equiv R'$ and we are done!
Attachments:
Z K Y
The post below has been deleted. Click to close.
This post has been deleted. Click here to see post.
Eeightqx
53 posts
#29
Y by
Below are all measured sections.
Let $G$ be the midpoint of $BC$, where we can know the midpoint of $DE$ is also $G$. Point $R\in AG$ s.t.
$$\dfrac{AR}{RG}=\dfrac23.$$
From
$$\dfrac{AM}{MD}\cdot\dfrac{DB}{BG}\cdot\dfrac{GR}{RA}=-1,$$by Menelaus' Thm. we kan get $B,\,M,\,R$ collinear. Similarly $C,\,N,\,R$ collinear.
Let $\odot(DHM),\,\odot(EHN)$ cut $BC$ again at $X,\,Y$ respectively.
From
$$DH=\dfrac{AE}{\sin\angle ADE}\cos\angle ADE,$$by a corollary of Ptolemy's Thm. we get
$$
\begin{aligned}
&DX\cos\angle DAE+DM\cos\angle AED=DH\sin\angle ADE\\
\Longleftrightarrow&DX=\dfrac{AE\cos\angle ADE-\dfrac12DA\cos\angle AED}{\cos\angle DAE}.
\end{aligned}
$$Similarly
$$EY=\dfrac{\dfrac12EA\cos\angle ADE-AD\cos\angle AED}{\cos\angle DAE}.$$So
$$DX+EY=\dfrac1{\cos\angle DAE}\cdot\dfrac32\left(AE\cos\angle ADE-AD\cos\angle AED\right)=\dfrac32\cdot\dfrac{AD^2-AE^2}{DE}.$$
Let $f(X)=Pow_{\odot(DHM)}(X)-Pow_{\odot(EHN)}(X)$, then $f$ is a linear function. Easy to see
$$f(A)=AM\cdot AD-AN\cdot AE=\dfrac12(AD^2-AE^2),$$and
$$f(G)=\dfrac12(f(D)+f(E))=\dfrac12(-DE\cdot DY+DE\cdot XE)=-\dfrac12DE(EX-DY)=-\dfrac12DE(DX+EY)=-\dfrac34(AD^2-AE^2).$$So
$$f(R)=\dfrac35f(A)+\dfrac25f(G)=0,$$That is $R$ is on the radical axis of $\odot(DHM)$ and $\odot(EHN)$.

So
$$RM\cdot RP=RN\cdot RQ,$$and we are done.
Z K Y
The post below has been deleted. Click to close.
This post has been deleted. Click here to see post.
Mahdi_Mashayekhi
694 posts
#30 • 1 Y
Y by sami1618
Let $BM,CN$ meet at $S$. Let $AS$ meet $BC,MN$ at $K,K'$ and $AH$ meet $BC,MN$ at $T,T'$. Let $MN$ meet $DHM$ and $EHN$ at $L,J$.
Claim $: AS$ bisects $BC$.
Proof $:$ by Menelaus on $ADK$ wrt $SM$ and $AEK$ wrt $SN$ we have $\frac{DB}{KB}=\frac{EC}{KC}$ so $BK=KC$.

this also proves $K$ is midpoint of $DE$ and $K'$ is midpoint of $MN$. Also since $\angle MLH = \angle MDH = \angle NEH = \angle NJH$ so $HLJ$ is isosceles and since $AH \perp MN$ then $T$ is midpoint of $LJ$. also note that $\frac{KS}{SA}=\frac{3}{2}$. Let $f(X)=Pow_{\odot(DHM)}(X)-Pow_{\odot(EHN)}(X)$ so we need to prove $f(S)=0$. by linearity of PoP we have that $f(S)=\frac{3}{5}f(A)+\frac{2}{5}f(K) = \frac{3}{5}f(A)+\frac{1}{5}f(D)+\frac{1}{5}f(E) = \frac{1}{5}f(A) + \frac{2}{5}(f(M)+f(N)) = \frac{1}{5}f(A) + \frac{2}{5}(-MN.(T'J+MK'+T'K')+NM.(T'L+NK'+K'T')) = \frac{1}{5}f(A)+\frac{4}{5}(MN.T'K') = \frac{1}{5}(f(A)+DE.TK) = \frac{1}{5}(\frac{AD^2-AE^2}{2}+DE.TK)$ which is well-known that $\frac{AD^2-AE^2}{2}+DE.TK=0$ so $f(S)=0$ as wanted.
Z K Y
The post below has been deleted. Click to close.
This post has been deleted. Click here to see post.
NuMBeRaToRiC
17 posts
#31 • 1 Y
Y by Electro47
A Beautiful Solution!
Let $T$ be a second intersection point of $(DHM)$ and $(EHN)$ and $O$ is the circumcenter of $(AMN)$. From easy angle chasing we get that quadrilateral $(MONT)$ cyclic. From cyclic we get that
$\angle MTO=\angle MNO=\angle HDA=180-\angle HTM$ so we get that $H, T, O $ collinear. Let $R$ be a intersection point of perpendicular line through $D$ to $AD$ and line $OM$. Similarly we define the point $S$. $\frac{DM}{DR}=\tan\angle AED=\frac{AD}{EH}=\frac{2DM}{EH}$ so we get that $EH=2DR$, from $BD=DE$ and $DR \parallel EH$ we get that $B, R, H$ collinear and $R$ is the midpoint of $BH$. Similarly $S$ is the midpoint of $CH$. So we get that $RS \parallel BC$.
From radical axis theorem we have to prove that lines $BM, CN$ and $OH$ concurrent. The points $OM\cap BH=R$, $ON\cap CH=S$ and $MN\cap BC={P}_\infty$ are collinear, so from Desarguess theorem we get that triangles $OMN$ and $HBC$ are perspective as desired. So we are done!
Z K Y
The post below has been deleted. Click to close.
This post has been deleted. Click here to see post.
GeoKing
518 posts
#32 • 5 Y
Y by hectorleo123, StefanSebez, sami1618, VicKmath7, Aryan27
Here's a video sol:- I have tried the explain the motivation behind the sol
https://youtu.be/o8q3-bka9Fg?si=WlR3DDYO3MRZgfSo
Z K Y
The post below has been deleted. Click to close.
This post has been deleted. Click here to see post.
juckter
323 posts
#33
Y by
Great problem!

Let $X$ and $Y$ be points such that $AEDX$ and $ADEY$ are paralellograms ($XD \parallel AE$, $AX \parallel DE$, $AD \parallel EY$, $AY \parallel DE$). Then $E, M, X$ and $D, N, Y$ are collinear triples. Notice that $ABDY$ and $ACEX$ are also parallelograms since $AY = BD$ and $AX = CE$. It follows that $B, N, Y$ and $C, M, X$ are also collinear.

Main Claim. Points $P$ and $Q$ lie on the circumcircle of $\triangle HXY$.

Proof. Let $H'$ be the reflection of $H$ about $M$. It is well known that $H'$ is the $E$-antipode in $(ADE)$ and $EH'$, $EH$ are isogonal with respect to $\angle AED$. Now angle chase

\begin{align*}
\measuredangle HPY = \measuredangle HPM = \measuredangle HDM = \measuredangle HDA = \measuredangle AEH = \measuredangle H'ED = \measuredangle HXA = \measuredangle HXY
\end{align*}
Where $\measuredangle H'ED = \measuredangle HXA$ follows from reflecting $H'$, $E$, and $D$ about $M$. Thus $P$ lies on the circumcircle of $\triangle HXY$. Analogously $Q$ lies on this circumcircle. $\square$

Finally, since $MN \parallel DE \parallel XY$ we have

\[\measuredangle MPQ = \measuredangle YPQ = \measuredangle YXQ = \measuredangle YXN = \measuredangle MNX = \measuredangle MNQ\]
And thus $P, Q, M, N$ are concyclic.
Z K Y
The post below has been deleted. Click to close.
This post has been deleted. Click here to see post.
kamatadu
478 posts
#34 • 2 Y
Y by L13832, SilverBlaze_SY
Solved with SilverBlaze_SY.

[asy]
/* Converted from GeoGebra by User:Azjps using Evan's magic cleaner https://github.com/vEnhance/dotfiles/blob/main/py-scripts/export-ggb-clean-asy.py */ /* A few re-additions are done using bubu-asy.py. This adds the dps, xmin, linewidth, fontsize and directions. https://github.com/Bubu-Droid/dotfiles/blob/master/bubu-scripts/bubu-asy.py */
pair A = (4.86240,33.99728); pair D = (-1.88462,-8.34117); pair E = (32.20988,-8.34117); pair B = (-35.97913,-8.34117); pair C = (66.30439,-8.34117); pair M = (1.48889,12.82805); pair N = (18.53614,12.82805); pair X = (8.98249,17.06190); pair T = (15.16262,-8.34117);
import graph; size(10cm); pen dps = linewidth(0.5) + fontsize(13); defaultpen(dps);
draw(A--D, linewidth(0.6)); draw(D--T, linewidth(0.6)+orange); draw(T--E, linewidth(0.6)+orange); draw(E--A, linewidth(0.6)); draw(D--B, linewidth(0.6)+red); draw(E--C, linewidth(0.6)+red); draw(A--B, linewidth(0.6)); draw(A--C, linewidth(0.6)); draw(B--X, linewidth(0.6)+blue); draw(X--C, linewidth(0.6)+blue); draw(A--T, linewidth(0.6));
dot("$A$", A, NW); dot("$D$", D, NW); dot("$E$", E, NE); dot("$B$", B, NW); dot("$C$", C, NE); dot("$M$", M, NW); dot("$N$", N, NE); dot("$X$", X, NW); dot("$T$", T, NE);
[/asy]

Let $X=BM\cap CN$ and $T=AX\cap DE$.

Then by applying Menelaus Theorem on $\triangle ADT$ with $MX$ as the transversal, we get,
\[
\frac{AM}{MD}\cdot \frac{DB}{BT}\cdot \frac{TX}{XA}=-1
\]which gives $\frac{DB}{BT}=-\frac{XA}{TX}$.

Similarly, we also get $\frac{EC}{CT}=-\frac{XA}{TX}$.

This means that $\frac{EC}{CT}=\frac{DB}{BT}$ which further implies that $DT=TE$, i.e., $T$ is the midpoint of $\triangle ADE$.

Now plugging this back into the first expression that we derived by applying Menelaus, we get that $\frac{AX}{XT}=\frac{2}{3}$.

[asy]
/* Converted from GeoGebra by User:Azjps using Evan's magic cleaner https://github.com/vEnhance/dotfiles/blob/main/py-scripts/export-ggb-clean-asy.py */ /* A few re-additions are done using bubu-asy.py. This adds the dps, xmin, linewidth, fontsize and directions. https://github.com/Bubu-Droid/dotfiles/blob/master/bubu-scripts/bubu-asy.py */ pair A = (4.86240,33.99728); pair D = (-1.88462,-8.34117); pair E = (32.20988,-8.34117); pair M = (1.48889,12.82805); pair N = (18.53614,12.82805); pair H = (4.86240,-3.98309); pair X = (8.98249,17.06190); pair O = (10.01251,22.32315); pair T = (15.16262,-8.34117); pair G = (6.77954,5.80944);
import graph; size(10cm); pen dps = linewidth(0.5) + fontsize(13); defaultpen(dps);
draw(A--D, linewidth(0.6)); draw(D--E, linewidth(0.6)); draw(E--A, linewidth(0.6)); draw(circle((-4.36993,2.90829), 11.52073), linewidth(0.6) + blue); draw(circle((19.24485,-1.71489), 14.56020), linewidth(0.6) + blue); draw(H--O, linewidth(0.6) + linetype("4 4") + red); draw(A--T, linewidth(0.6));
dot("$A$", A, NW); dot("$D$", D, dir(270)); dot("$E$", E, SE); dot("$M$", M, SW); dot("$N$", N, NE); dot("$H$", H, NW); dot("$X$", X, NW); dot("$O$", O, NW); dot("$T$", T, NE); dot("$G$", G, NW);
[/asy]

In order to prove that $MNPQ$ is cyclic, it suffices to show that $X$ lies on the radical axis of $\left\{ \odot(DHM),\odot(EHN) \right\}$.

Let $G$ denote the second intersection of $\odot(DHM)$ and $\odot(EHN)$. Also, $O$ be the center of $\odot(AMN)$.

Then,
\[ \measuredangle NGM=\measuredangle HGM+\measuredangle NGH =\measuredangle HDM+\measuredangle NEH =2(90^{\circ}-\measuredangle DAE) =\measuredangle NOM \]which implies that $OMGN$ is cyclic. Furthermore,
\[ \measuredangle NGO=\measuredangle NMO =90^{\circ}-\measuredangle MAN =\measuredangle AEH=\measuredangle NEH=\measuredangle NGH \]which gives us that $\overline{O-G-H}$ are collinear.

To finally show that $X$ lies on the radical axis, note that it is enough to show that $\overline{O-X-H}$ are collinear.

Claim: $\overline{O-X-H}$ are collinear.
Proof. We use complex number to prove this. We denote the affixes of the points with the smaller case of their labels.

Consider $\triangle AMN$ to be on the unit circle. Then note that $o = 0$. Let $h'$ denote the affix of the orthocenter of $\triangle AMN$. Then note that a dilation centered at $A$ with scale $2$ sends $H'$ to $H$. Clearly $h'=a+m+n$.

Shifting $A$ to the origin, scaling by a factor of $2$ and then shifting back gives that $h=2m+2n+a$.

Also, as we had found out that $\frac{AX}{XT}=\frac{2}{3}$, by section formula we can derive that,
\[ x=\frac{3a+2t}{5}=\frac{3a+d+e}{5} =\frac{3a+(2n-a)+(2m-a)}{5}=\frac{a+2n+2m}{5} .\]
In order to show that $\overline{O-X-H}$ are collinear, it suffices to show that,
\[ \frac{h}{x}=5\cdot \frac{2m+2n+a}{a+2n+2m} =5\in\mathbb{R} \]which is clearly true and our claim is proved. $\blacksquare$

Using our claim, we can conclude that $\overline{O-X-G-H}$ are collinear and we are done.
This post has been edited 1 time. Last edited by kamatadu, Apr 17, 2025, 4:50 PM
Z K Y
The post below has been deleted. Click to close.
This post has been deleted. Click here to see post.
breloje17fr
37 posts
#35 • 1 Y
Y by ehuseyinyigit
Please note the initial problem can be generalized by defining D and E as symetric points with respect to the midpoint of BC and points M and N on AD and AE respectively such that MN is parallel to BC.
Attachments:
This post has been edited 1 time. Last edited by breloje17fr, Apr 19, 2025, 1:58 PM
Reason: typing mistake
Z K Y
The post below has been deleted. Click to close.
This post has been deleted. Click here to see post.
Assassino9931
1251 posts
#36
Y by
Proposed by GeoGen
Z K Y
The post below has been deleted. Click to close.
This post has been deleted. Click here to see post.
dangerousliri
928 posts
#37 • 1 Y
Y by Assassino9931
This problem was proposed by Slovakia.
Z K Y
The post below has been deleted. Click to close.
This post has been deleted. Click here to see post.
v_Enhance
6876 posts
#38 • 1 Y
Y by GeoKing
Solution from Twitch Solves ISL:

We proceed by barycentric coordinates on $\triangle ADE$. Let $a = DE$, $b = EA$, $c = AD$. Recall $H = (S_B S_C : S_C S_A : S_A S_B)$. Finally, let $T$ be the intersection of lines $BM$ and $CN$.

[asy]
/* Converted from GeoGebra by User:Azjps using Evan's magic cleaner https://github.com/vEnhance/dotfiles/blob/main/py-scripts/export-ggb-clean-asy.py */ pair A = (-2.5,1.5); pair D = (-3.42948,-1.49748); pair E = (0.,-1.5); pair B = (-6.85897,-1.49496); pair C = (3.42948,-1.50251); pair H = (-2.50163,-0.72426); pair M = (-2.96474,0.00125); pair N = (-1.25,0.); pair T = (-2.18589,0.30050); pair P = (-4.01792,-0.40338); pair Q = (-0.59976,-0.20878);
size(11cm); pen yqqqyq = rgb(0.50196,0.,0.50196); pen zzttqq = rgb(0.6,0.2,0.); pen cqcqcq = rgb(0.75294,0.75294,0.75294); draw(A--D--E--cycle, linewidth(0.6) + zzttqq);
draw(circle((-3.29251,-0.71852), 0.79090), linewidth(0.6) + yqqqyq); draw(circle((-1.31746,-1.32705), 1.32876), linewidth(0.6) + yqqqyq); draw(A--D, linewidth(0.6) + zzttqq); draw(D--E, linewidth(0.6) + zzttqq); draw(E--A, linewidth(0.6) + zzttqq); draw(B--D, linewidth(0.6) + gray); draw(C--E, linewidth(0.6) + gray); draw(B--T, linewidth(0.6) + gray); draw(C--T, linewidth(0.6) + gray);
dot("$A$", A, dir((2.269, 5.764))); dot("$D$", D, dir((-7.595, -25.118))); dot("$E$", E, dir((4.622, -19.609))); dot("$B$", B, dir((-5.791, -19.529))); dot("$C$", C, dir((-5.942, -20.526))); dot("$H$", H, dir((5.937, -8.391))); dot("$M$", M, dir((-19.603, 16.026))); dot("$N$", N, dir((2.278, 4.468))); dot("$T$", T, dir((2.402, 4.794))); dot("$P$", P, dir((-10.671, 6.253))); dot("$Q$", Q, dir((2.095, 4.902)));
[/asy]


Claim: We have $T = (3:1:1)$.
Proof. Write $B = (0:2:-1)$ and $M = (1:1:0)$. Note that \[ \det \begin{bmatrix} 3 & 1 & 1 \\ 1 & 1 & 0 \\ 0 & 2 & -1 \\ \end{bmatrix} \]so $T$ lies on $BM$. $\blacksquare$

Claim: Point $T$ lies on the radical axis of $(DMH)$ and $(ENH)$.
Proof.
The circumcircle of $(DMH)$ then has equation given by \[ 0 = -a^2yz - b^2zx - c^2xy + (x+y+z)\left( \frac{c^2}{2} x + w z \right) \]for some constant $w$ (by plugging in $D$ and $H$). To determine $w$, plug in $H$ to get \begin{align*} \frac{c^2}{2} S_B S_C + w \cdot S_A S_B &= S_A S_B S_C \cdot \frac{a^2S_A + b^2S_B + c^2S_C}{S_B S_C + S_C S_A S_A S_B} = S_A S_B S_C \cdot \frac{8[ABC]^2}{4[ABC]^2} \\ \implies \frac{c^2}{2} S_C + S_A \cdot w &= 2 S_A S_C \\ \implies S_A \cdot w &= S_C \left(b^2 + \frac{1}{2} c^2-a^2 \right). \end{align*}In other words, $(DMH)$ has equation given by \[ 0 = -a^2yz - b^2zx - c^2xy + (x+y+z)\left( \frac{c^2}{2} x + \frac{S_C(b^2+c^2/2-a^2)}{S_A} z \right). \]Similarly, $(CNH)$ has equation given by \[ 0 = -a^2yz - b^2zx - c^2xy + (x+y+z)\left( \frac{b^2}{2} x + \frac{S_B(c^2+b^2/2-a^2)}{S_A} y \right). \]To check $T = (3:1:1)$ lies on the radical axis, it suffices to check equality of the linear parts, that is: \[ \frac{c^2}{2} \cdot 3 + \frac{S_C (b^2+c^2/2-a^2)}{S_A} = \frac{b^2}{2} \cdot 3 + \frac{S_B (c^2+b^2/2-a^2)}{S_A}. \]Using a common denominator for the left-hand side, we get \begin{align*} \frac{c^2}{2} \cdot 3 + \frac{S_C (b^2+c^2/2-a^2)}{S_A} &= \frac{3c^2S_A + S_C(2b^2+c^2-2a^2)}{2S_A} \\ &= \frac{3c^2(b^2+c^2-a^2) + (a^2+b^2-c^2)(2b^2+c^2-2a^2)}{2S_A} \\ &= \frac{b^4+b^2c^2+c^4-a^4}{S_A}. \end{align*}Since this is symmetric in $b$ and $c$, we're done. $\blacksquare$
Since $T$ lies on the radical axis, it follows that $TM \cdot TP = TN \cdot TQ$ and the problem is solved.
Z K Y
The post below has been deleted. Click to close.
This post has been deleted. Click here to see post.
G81928128
13 posts
#39 • 3 Y
Y by trigadd123, mofumofu, Assassino9931
I coordinated this problem at EGMO! Here are two solutions that I came up with during testsolving.

Reduction
Solution 1 (coordinates)
Solution 2 (slightly unhinged synthetic)

My blog post details the whole thought process.
Z K Y
N Quick Reply
G
H
=
a